[Boards: 3 / a / aco / adv / an / asp / b / bant / biz / c / can / cgl / ck / cm / co / cock / d / diy / e / fa / fap / fit / fitlit / g / gd / gif / h / hc / his / hm / hr / i / ic / int / jp / k / lgbt / lit / m / mlp / mlpol / mo / mtv / mu / n / news / o / out / outsoc / p / po / pol / qa / qst / r / r9k / s / s4s / sci / soc / sp / spa / t / tg / toy / trash / trv / tv / u / v / vg / vint / vip / vp / vr / w / wg / wsg / wsr / x / y ] [Search | Free Show | Home]

Consider this equation: A^x+B^y=C^z Where A,B,C,x,y, and z

This is a blue board which means that it's for everybody (Safe For Work content only). If you see any adult content, please report it.

Thread replies: 42
Thread images: 4

Consider this equation:

A^x+B^y=C^z

Where A,B,C,x,y, and z are positive integers, x,y, and z being more than 2, and the largest common divisor of A,B, and C being 1.

Can this equation exist, and if it is impossible, why?
>>
go to bed wiles
>>
This equation cannot exist. You didn't actually write anything down.
>>
There aren't any solutions if x, y, and z are equal. That's a start.
>>
1^ 5 + 2^3 = 3^2
>>
>>7714977
Not even the answer can have a power of 2
>>
>>7714945
Because of Fermat's Last Theorem.
>>
0^4 + 3^2 = 9^1
>>
Wow this is worth a million bucks. https://en.m.wikipedia.org/wiki/Beal%27s_conjecture
>>
>>7714864
I think when you put that many constraints on it it becomes pointless.
>>
Ever permutation of 2,4,4 for x,y, and z is wrong because of this proof. https://en.m.wikipedia.org/wiki/Proof_by_infinite_descent#Non-solvability_of_r2_.2B_s4_.3D_t4
>>
>>7714983
>>7714977
Are you guys even able to read?
>>
>>7714998
See >>7714992

Lots of "functions," (infinitely many) have no solutions over R or C. Big fucking deal if this one doesn't.
>>
Every permutation of n,4,4 for x,y, and z is wrong because of this proof. http://www.staff.science.uu.nl/~beuke106/Fermatlectures.pdf
>>
>>7714864
>filename
wat?
>>
>>7714871
this
>>
x,y, and z being 7 or less with A,B, and C being 250,000 or less has no results. x,y, and z being 100 or less with A,B, and C being 10,000 or less has no results.
>>
>>7714864
[math] 2^3 + 2^3 = 2^4 [/math]

Prove by example.
>>
>>7715055
>and the largest common divisor of A,B, and C being 1.
>>
File: doh.gif (14KB, 581x600px) Image search: [Google]
doh.gif
14KB, 581x600px
>>7715058
>>
File: 1449538579709.jpg (46KB, 473x500px) Image search: [Google]
1449538579709.jpg
46KB, 473x500px
>>7715059
>>
>>7715055
2 goes into 2.
>>
5^3 = 7^2 + [sqrt(76)]^2

/thread
>>
File: hisv.jpg (241KB, 1155x897px) Image search: [Google]
hisv.jpg
241KB, 1155x897px
>>7715070
>>
>>7714864

Not possible, I've tried.
>>
The case (x, y, z) = (2, 3, 7) and all its permutations were proven to have only four solutions, none of them involving an even power greater than 2. http://arxiv.org/pdf/math/0508174v1.pdf
>>
14+13=14
>>
>>7715236
Wooops 1^4+1^3=1^4
>>
>>7715238
since when does 1+1 = 1?
>>
>>7715236
>>7715238
two wrongs do not make a right
>>
1^9+1^7=sqrt(2)^2?
>>
>>7715298
Nevermind. Integers.
>>
>>7715070
Larger than two
>>
>>7715256
Boole.
>>
>>7715256
Boolean expression faggot
>>
The case (x, y, z) = (2, 3, 8) and all its permutations are known to have only three solutions, none of them involving an even power greater than 2 because of this proof http://www.staff.science.uu.nl/~beuke106/Fermatlectures.pdf
>>
>>7714945
Not exactly. For x = y = z = n and a, b, and c all real numbers, there does not exist a set of values that satisfy the function such that n >=3
>>
>a^x
I may be an autistic newfag, but doesn't /sci/ have some sort of plugin or tag similar to the [code] tag on /g/ that makes it so you can write superscripts properly?
>>
>>7715995
>all real numbers
Are you retarded?
0^n + 0^n = 0^n for all n>= 3.
>>
>>7714992
It doesn't, though I understand how someone could think that. Any time you've answered a bunch of questions about something, the questions about that thing become more complex. This is just something a lot of people have asked questions about.>>7715004
Because it's interesting. If that's not enough for you, look up Andrew Wiles and Fermat's Last Theorem. A solution to a question similar to this actually had far reaching consequences in various fields.
>>
>>7714864
Well, it can exist. It may never be valid under the parameters given, but
>>
>>7715004
>Big fucking deal
That's right, Cletus:
>>7714988
Thread posts: 42
Thread images: 4


[Boards: 3 / a / aco / adv / an / asp / b / bant / biz / c / can / cgl / ck / cm / co / cock / d / diy / e / fa / fap / fit / fitlit / g / gd / gif / h / hc / his / hm / hr / i / ic / int / jp / k / lgbt / lit / m / mlp / mlpol / mo / mtv / mu / n / news / o / out / outsoc / p / po / pol / qa / qst / r / r9k / s / s4s / sci / soc / sp / spa / t / tg / toy / trash / trv / tv / u / v / vg / vint / vip / vp / vr / w / wg / wsg / wsr / x / y] [Search | Top | Home]

I'm aware that Imgur.com will stop allowing adult images since 15th of May. I'm taking actions to backup as much data as possible.
Read more on this topic here - https://archived.moe/talk/thread/1694/


If you need a post removed click on it's [Report] button and follow the instruction.
DMCA Content Takedown via dmca.com
All images are hosted on imgur.com.
If you like this website please support us by donating with Bitcoins at 16mKtbZiwW52BLkibtCr8jUg2KVUMTxVQ5
All trademarks and copyrights on this page are owned by their respective parties.
Images uploaded are the responsibility of the Poster. Comments are owned by the Poster.
This is a 4chan archive - all of the content originated from that site.
This means that RandomArchive shows their content, archived.
If you need information for a Poster - contact them.